0

Logarithmic notation - class-XI

Description: logarithmic notation
Number of Questions: 48
Created by:
Tags: real numbers physics maths elementary mathematics logarithms basic algebra basic mathematical concepts
Attempted 0/47 Correct 0 Score 0

If $\log\ (-2x)=2\log\ (x+1)$, then $x$ can be  equal to

  1. $-2+\sqrt {3}$

  2. $-4+2\sqrt {3}$

  3. $-2-\sqrt {3}$

  4. $-4-2\sqrt {3}$


Correct Option: A,C
Explanation:

We have,

$ \log \left( -2x \right)=2\log \left( x+1 \right) $

$ \Rightarrow \log \left( -2x \right)=\log {{\left( x+1 \right)}^{2}} $


Comparing both side and we get,

$ -2x={{\left( x+1 \right)}^{2}} $

$ \Rightarrow -2x={{x}^{2}}+1+2x $

$ \Rightarrow {{x}^{2}}+4x+1=0 $


Using quadratic formula and we get,

$ x=\dfrac{-4\pm \sqrt{16-4\times 1\times 1}}{2\times 1} $

$ x=\dfrac{-4\pm \sqrt{12}}{2} $

$ x=\dfrac{-4\pm \sqrt{2\times 2\times 3}}{2} $

$ x=\dfrac{-4\pm 2\sqrt{3}}{2} $

$ x=-2\pm \sqrt{3} $


Hence, this is the answer.

If $\displaystyle \log _3 x = 0$, then value of $x$ is equal to

  1. $2$

  2. $4$

  3. $1$

  4. $3$


Correct Option: C
Explanation:
$\log _3x = 0$      

$x=3^0$   ....(since we know that $\log _ab=t\Rightarrow b=a^t$)
     
$\therefore x=1$

State true or false: 

If$\displaystyle x^y = z$, then $\displaystyle y = log _z x$.

  1. True

  2. False


Correct Option: B
Explanation:

$x^y=z$
Apply log on both the sides, we get
$\therefore ylogx=logz$
$\therefore y=\dfrac{logz}{logx}$
$\therefore y=log _xz$
Hence, it is false.

The value of $\log _{10}0.01$ is equal to 

  1. $0$

  2. $-2$

  3. $-1$

  4. $4$


Correct Option: B
Explanation:

$\log _{ 10 }{ 0.01= } \log _{ 10 }{ { 10 }^{ -2 } } $

$\log _{ 10 }{ 0.01= } -2$
Answer (B) -2

The exponential form of $\log _{10}1 = 0$ is $10^{m} = 1$,  then the value of $m$ is 

  1. $2$

  2. $0$

  3. $1$

  4. $6$


Correct Option: B
Explanation:

$ \log _{ 10 }{ 1 }= 0\ 1 = { 10 }^{ 0 }$

The value of $\log _5\ 125$ is equal to

  1. $0$

  2. $1$

  3. $2$

  4. $3$


Correct Option: D
Explanation:

$ \log _{ 5 }{ 125 } =x\ 125 = { 5 }^{ x }\ x = 3 $

The value of $\log _5 1$ is equal to

  1. $1$

  2. $0$

  3. $7$

  4. $2$


Correct Option: B
Explanation:

$ \log _{ 5 }{ 1 } =x\ 1\quad =\quad { 5 }^{ x }\ x\quad =\quad 0\ \ $

If exponential form of $\log _{10} 0.01 = -2$ is $10^{m} = 0.01$, then value of $m$ is equal to

  1. $-1$

  2. $3$

  3. $-2$

  4. $4$


Correct Option: C
Explanation:

$ \log _{ 10 }{ 0.01 } = -2 \Rightarrow 0.01 = { 10 }^{ -2 } $

$\therefore m=-2$

Express the following in logarithmic form$\,\colon$
$81\,=\,3^{4}$

  1. $\log _381\,=\,4$

  2. $\log _981\,=\,2$

  3. $2\log _39\,=\,4$

  4. $4\log _93\,=\,2$


Correct Option: A,B,C,D
Explanation:

$y={ a }^{ x }\Rightarrow \log _{ a }{ y } =x\ \therefore 81={ 3 }^{ 4 }\Rightarrow \log _{ 3 }{ 81 } =4$

A is true.
$81=3^{4}=9^{2}$

$\Rightarrow \log _{9}81=\log _{9}9^{2}=2\log _{9} 9=2$
B is true.
$81=3^{4}=9^{2}$
$\Rightarrow \log _{3} 3^{4}=\log _{3}9^{2}=2\log _{3} 9$
C is true.
$81=3^{4}=9^{2}$
$\Rightarrow \log _{9}3^{4}=\log _{9}9^{2}=2\log _{9} 9=2$
$\Rightarrow 4\log _{9} 3=2$
D is true.

If $log 27 = 1.431$, then the value of $log 9$ is

  1. 0.934

  2. 0.945

  3. 0.954

  4. 0.958


Correct Option: C
Explanation:

$log 27 = 1.431$
$\Rightarrow log (3^3) = 1.431$
$\Rightarrow 3 log 3 = 1.431$
$\Rightarrow log 3 = 0.477$
$\therefore log 9 = log (3^2) = 2 log 3 = (2 \times 0.477) = 0.954$

Find the correct expression, if $\log _{ c }{ a } =x$.

  1. ${ a }^{ c }=x$

  2. ${ a }^{ x }=c$

  3. ${ c }^{ a }=x$

  4. ${ c }^{ x }=a$

  5. ${ x }^{ c }=a$


Correct Option: D
Explanation:

Given, $\log _c a=x$

We know the change of base formula:
$\log _c a = x$  is $c^x = a$

Which of the following statements is not correct?

  1. $log _{10} 10 = 1$

  2. $log (2+ 3) = log (2 \times 3)$

  3. $log _{10} 1 = 0$

  4. $log (1 + 2 + 3) = log 1 + log 2 + log 3$


Correct Option: B
Explanation:

(a) Since $log _a a = 1,$ so $log _{10}  10 = 1$
(b) $log (2 + 3) log 5$ and $log (2 \times 3) = log 6 = log 2 + log 3$
$\therefore log(2 + 3) \neq log (2 \times 3)$
(c) Since, $log _a  1 = 0$, so, $log _{10} 1 = 0$.
(d) $log(1 + 2 + 3) = log 6 = log (1 \times 2 \times 3) = log 1 + log 2 + log 3$

If $log _{10} 2 = 0.3010$, the value of $log _{10}$ 80 is

  1. 1.6020

  2. 1.9030

  3. 3.9030

  4. None of these


Correct Option: B
Explanation:

$log _{10} 80 = log _{10} (8 \times 10)$
$= log _{10} 8 + log _{10} 10$
$=log _{10} (2^3) + 1$
$= 3 log _{10} 2 + 1$
$= (3 \times 0.3010) + 1$
$= 1.9030$

If $log 2 = 0.3010 $ and $3 = 0.4771$, the value of $log _5 512$ is

  1. 2.870

  2. 2.967

  3. 3.876

  4. 3.912


Correct Option: C
Explanation:

$log _5 512 = \dfrac{log 512}{log 5}$
$=\dfrac{log 2^9}{log (10/2)}$
$=\dfrac{9 log  2}{log 10 - log 2}$
$=\dfrac{9 \times 0.3010}{1- 0.3010}$
$= \dfrac{2.709}{0.699}$
$=\dfrac{2709}{699}$
$= 3.876$

If $log 2 = 0.30103$, the number of digits in $2^{64}$ i

  1. 18

  2. 19

  3. 20

  4. 21


Correct Option: C
Explanation:

$log (2^{64}) = 64 \times log 2$
$= (64 \times 0.30103)$
$= 19.2592$
Its characteristic is 19.
Hence, then number of digits in $2^{64}$ is 20.

If $log _x \left( \dfrac{9}{16} \right) = - \dfrac{1}{2}$, then x is equal to

  1. $- \dfrac{3}{4}$

  2. $\dfrac{3}{4}$

  3. $\dfrac{81}{256}$

  4. $\dfrac{256}{81}$


Correct Option: D
Explanation:

$log _x \left( \dfrac{9}{16} \right ) = - \dfrac{1}{2}$
$\Rightarrow x^{-1/2} = \dfrac{9}{16}$
$\Rightarrow \dfrac{1}{\sqrt x} = \dfrac{9}{16}$
$\Rightarrow \sqrt x = \dfrac{16}{9}$
$\Rightarrow x = \left( \dfrac{16}{9} \right)^2$
$\Rightarrow x = \dfrac{256}{81}$

What is the value of $\dfrac {1}{2}\log _{10} 25 - 2 \log _{10} 3 +\log _{10} 18$?

  1. $2$

  2. $3$

  3. $1$

  4. $0$


Correct Option: C
Explanation:

The value of $\dfrac {1}{2}\log _{10} 25 - 2 \log _{10} 3 +\log _{10} 18$ is
$= \log _{10}(25)^{1/2} - \log _{10} (3)^{2} + \log _{10}18$
$= \log _{10}5 - \log _{10}9 + \log _{10}18$
$= \log _{10} \left (\dfrac {5}{9}\times 18\right ) $

$= \log _{10} 10 $    ....Using the identity $\log _aa=1$
$= 1$

The value of $log _2$ 16 is

  1. $\dfrac{1}{8}$

  2. 4

  3. 8

  4. 16


Correct Option: B
Explanation:

Let $log _2      16 = n$
Then, $2^n = 16 = 2^4    \Rightarrow n = 4$
$\therefore log _2  16 = 4$

The logarithmic form of ${5}^{2}=25$ is

  1. $\log _{ 5 }{ 2 } =25$

  2. $\log _{ 2 }{ 5 } =25$

  3. $\log _{ 5 }{ 25 } =2$

  4. $\log _{ 25 }{ 5 } =2$


Correct Option: C
Explanation:

$5^2=25$

Taking log with base $5$ both sides, we get
$\log _55^2=\log _525$
$\Rightarrow \log _525=2\log _55$
$\Rightarrow \log _525=2$     $(\log _aa=1)$
Hence, C is the correct option.

The exponential form of $\log _{ 2 }{ 16 } =4$ is

  1. ${2}^{4}=16$

  2. ${4}^{2}=16$

  3. ${2}^{16}=4$

  4. ${4}^{16}=2$


Correct Option: A
Explanation:

Exponential form of $\log _2 16=4$

Taking antilog both sides, we get
$\Rightarrow 16=2^4$
Hence, A is the correct option.

If mantissa of logarithm of 719.3 to the base 10 is 0.8569 , then mantissa of logarithm  of 71.93 is

  1. 0.8569

  2. $\overline 1 .8569$

  3. 1.8569

  4. 0.1431


Correct Option: A
Explanation:

Mantissa of logarithm of $719.3$ to base $10$ is $0.8569$

Then, mantissa of logarithm of $71.93$ is also $0.8569$
As, $\log _{10}{(719.3)}=2+(0.8569)$(mantissa)
So, $\log _{10}{(71.93)}=1+(0.8596)$ (mantissa)

If $2\log y -\log x -3=0$, express $x$ in terms of $y.$

  1. $x=\dfrac{y^2}{e^3}$

  2. $x=\dfrac{y^2}{e^2}$

  3. $x^2=\dfrac{y^2}{e^3}$

  4. $x=\dfrac{y^3}{e^3}$


Correct Option: A
Explanation:
$\Rightarrow$$\log { { y }^{ 2 } } -\log { x } -\log { { e }^{ 3 } } =0$.......$\log e=1$

$\Rightarrow$$ \log { x } =\log { \left (\cfrac { { y }^{ 2 } }{ { e }^{ 3 } } \right ) } $

$\Rightarrow$$ x=\cfrac { { y }^{ 2 } }{ { e }^{ 3 } } $

If $2\log y -\log x-3=0$ express $x$ in terms of $y.$

  1. $x^2=1000y$

  2. $x^2= \dfrac{y^2}{e^3}$

  3. $y^2= \dfrac{x}{1000}$

  4. $y^2= 1000x$


Correct Option: B
Explanation:
Given: $2\log y -\log x-3=0$

$\log { { y }^{ 2 } } -\log { x } -3\log { { e }=0 } $.......$(\log e=1)$

$\log { { y }^{ 2 } } -\log { x } -\log { { e }^{ 3 }=0 } $

$ \log { x } =\log { { y }^{ 2 } } -\log { { e }^{ 3 } } =\log { \left (\cfrac { { y }^{ 2 } }{ { e }^{ 3 } } \right ) } $

$ x=\cfrac { { y }^{ 2 } }{ { e }^{ 3 } } $

If $2x^{{log _4}^3}+3^{\log _4x}=27$, then x is equal to?

  1. $2$

  2. $4$

  3. $8$

  4. $16$


Correct Option: D
Explanation:
$2\times { x }^{ { log } _{ 4 }3 }+{ 3 }^{ { log } _{ 4 }x }=27$

${ 2\times3 }^{ { log } _{ 4 }x }+{ 3 }^{ { log } _{ 4 }x }=27$

Let ${ 3 }^{ { log } _{ 4 }x }=t$
$2t+t=27$
$3t=27$
$t=9$

${ 3 }^{ { log } _{ 4 }^{ x } }={ 3 }^{ 2 }=9$

So,  ${ log } _{ 4 }x=2$

So,  $x={ 2 }^{ 4 }=16$

$\log _{ 4 }{ 18 } $ is

  1. A rational number

  2. An irrational number

  3. A prime number

  4. None of these


Correct Option: B
Explanation:
$log _418=\dfrac{log18}{log4}$
$=\dfrac{log2+log9}{2log2}$
$=\dfrac{log2+2log3}{2log2}$
$=\dfrac{log2}{2log2}+\dfrac{2log3}{2log2}$
$=\dfrac{1}{2}+log _23$
Since $log _23$ is irrational
Hence $=\dfrac{1}{2}+log _23$ is irrational.
Therefore
$log _418$ is irrational number

The value of x, for which the 6th term in the expansion of $\left{ { 2 }^{ { log } _{ 2 }\sqrt { \left( { 9 }^{ x-1 }+7 \right)  }  }+\dfrac { 1 }{ { 2 }^{ { \left( 1/5 \right) log } _{ 2 }\left( { 3 }^{ x-1 }+1 \right)  } }  \right} ^{ 7 }$ is 84, is equal to 

  1. 4

  2. 3

  3. 2

  4. 1


Correct Option: C

If x = ${ log } _{ 3 }243,y={ log } _{ 2 }64,$, Then $\sqrt { x-2\sqrt { y }  } $ is 

  1. $\sqrt { 5-2\sqrt6 }$

  2. $2-\sqrt { 3 } $

  3. $\sqrt { 3 } -\sqrt { 2 } $

  4. $\sqrt { 3 } -4$


Correct Option: A
Explanation:

We have,

$ x={{\log } _{3}}243\,\,......\,\,\left( 1 \right) $

$ y={{\log } _{2}}64\,\,......\,\,\left( 2 \right) $

Applying rule,

${{\log } _{b}}x=y\,\Rightarrow {{b}^{y}}=x$

So,

From equation (1) and (2) to,

$ x={{\log } _{3}}243 $

$ \Rightarrow {{3}^{x}}=243 $

$ \Rightarrow {{3}^{x}}={{3}^{5}} $

$ \Rightarrow x=5 $

Now,

$ {{\log } _{2}}64=y $

$ \Rightarrow {{2}^{y}}=64 $

$ \Rightarrow {{2}^{y}}={{2}^{6}} $

$ \Rightarrow y=6 $

Now,

$ \sqrt{x-2\sqrt{y}} $

$ =\sqrt{5-2\sqrt{6}} $

Hence, this is the answer.

Logarithmic form of  $3 \sqrt { 8 } = 2$  is

  1. $\log _ { 8 } 2 = \dfrac { 1 } { 3 }$

  2. $\log _ { 2 } 8 = \dfrac { 1 } { 3 }$

  3. $\log _ { \frac { 1 } { 3 } } 8 = 2$

  4. $\log _ { \frac { 1 } { 3 } } 2 = 0$


Correct Option: A

Number of solutions of $\log _{4}{\left(x-1\right)}=\log _{2}{\left(x-3\right)}$

  1. $1$

  2. $2$

  3. $3$

  4. $4$


Correct Option: B
Explanation:
$\log _{4}{\left(x-1\right)}=\log _{2}{\left(x-3\right)}$
$\Rightarrow\,\log _{{2}^{2}}{\left(x-1\right)}=\log _{2}{\left(x-3\right)}$
$\Rightarrow\,\dfrac{1}{2}\log _{2}{\left(x-1\right)}=\log _{2}{\left(x-3\right)}$
$\Rightarrow\,\log _{2}{\left(x-1\right)}=2\log _{2}{\left(x-3\right)}$
$\Rightarrow\,\left(x-1\right)={\left(x-3\right)}^{2}$
$\Rightarrow \,x-1={x}^{2}-6x+9$
$\Rightarrow \,{x}^{2}-6x-x+9+1=0$
$\Rightarrow \,{x}^{2}-7x+10=0$
$\Rightarrow \,{x}^{2}-2x-5x+10=0$
$\Rightarrow \,x\left(x-2\right)-5\left(x-2\right)=0$
$\Rightarrow \,\left(x-2\right)\left(x-5\right)=0$
$\therefore\,x=2,\,5$
Number of solutions$=2$

The value of x, which satisfies the equation $2 \log _ { 2 } \left( \log _ { 2 } x \right) + \log _ { 12 } \left( \log _ { 2 } ( 2 \sqrt { 2 } x ) \right) = 1$ is greater

  1. 10

  2. 11

  3. 7

  4. 9


Correct Option: A

The logarithm form of $\displaystyle 5^3 = 125$ is equal to

  1. $\displaystyle \log _5 125 = 3$

  2. $\displaystyle \log _5 125 = 5$

  3. $\displaystyle \log _3 125 = 5$

  4. $\displaystyle \log _5 3 = 3$


Correct Option: A
Explanation:

$5^3=125$
Taking log on both sides, we get
$3\log 5 = \log 125$
$\log _5 125 = 3$         ...(since $\dfrac{\log a}{\log b} = \log _ba$)

The logarithmic form of $\displaystyle (81)^{\frac {3}{4}} = 27$ is

  1. $\displaystyle \log _{66} 36 = \frac {2}{9}$

  2. $\displaystyle \log _{81} 27 = \frac {3}{4}$

  3. $\displaystyle \log _{16} 33 = \frac {7}{2}$

  4. $\displaystyle \log _{78} 12 = \frac {1}{3}$


Correct Option: B
Explanation:

$81^{\frac{3}{4}}=27$
Taking log on both sides
$\dfrac{3}{4}log81=log27$
$log _{81}27= \dfrac{3}{4}$.....(since $\dfrac{loga}{logb}=log _ba$)

Given $\displaystyle 3^{x} = \frac {1}{9}$ then $x=?$

  1. $-1$

  2. $-2$

  3. $1$

  4. $2$


Correct Option: B
Explanation:

$3^{-2}=\dfrac{1}{9}$
Taking log on both sides
$-2log3= log\dfrac{1}{9}$
$log _3\dfrac{1}{9}= -2$.....(since $\dfrac{loga}{logb}=log _ba$)

Express in logarithmic form and find x: $\displaystyle 10^{x} = 0.001$ (i.e base 10)

  1. $3$

  2. $-3$

  3. $2$

  4. $-2$


Correct Option: B
Explanation:

$10^{-3}=0.001$
Taking log on both sides
$-3log10= log0.001$
$log _{10}0.001= -3$.....(since $\dfrac{loga}{logb}=log _ba$)

State whether true or false:
$\displaystyle \log : V = 2 \log : 2 - \log : 3 + \log : \pi + 3 \log : r$ gives $\displaystyle V = \frac {4}{3} \pi r^3$

  1. True

  2. False


Correct Option: A
Explanation:

$\displaystyle \log : V = 2 : \log : 2 - : \log : 3 + : \log : \pi + 3 : \log : r$
$\therefore \log V = \log 2^2-\log 3+\log \pi+ \log r$....($\log a^b=b \log a$)
$\therefore \log V= \log \cfrac{4}{3}\pi r^3$.....($\log a.b = \log a + \log b, \log \cfrac {a}{b} = \log a - \log b$)
$\therefore V=\cfrac{4}{3}\pi r^3$

The logarithm form of $10^{-3} = 0.001$ is $\log _{10} 0.001 = -m$, then value of $m$ is 

  1. $-1$

  2. $-2$

  3. $3$

  4. $-4$


Correct Option: C
Explanation:
The logarithm of the $10^{-3}=\frac{1}{1000}=0.0001$
So $log _{10}0.0001=-3$
$log _{10}0.0001=-m$
Then m=3

The value of $\displaystyle \log _{10}0.001 $ is equal to

  1. $-3$

  2. $3$

  3. $-2$

  4. $2$


Correct Option: A
Explanation:

$ \ \log _{ 10 }{ 0.001 } =x\ 0.001 = { 10 }^{ x }\ \dfrac { 1 }{ 1000 } = { 10 }^{ -3 }\ x=-3  $

The value of $\log _{0.5}16$ is equal to

  1. $-4$

  2. $-1$

  3. $-2$

  4. $0$


Correct Option: A
Explanation:

$ \log _{ 0.5 }{ 16 } = x $


$ 16 = {0.5 }^{ x }$

$ { 2 }^{ 4 } = { 2 }^{ -1x }$

$ x = -4\ \ 
$

The logarithm of $0.001$ to the base $10$ is equal to

  1. $5$

  2. $-1$

  3. $6$

  4. $-3$


Correct Option: D
Explanation:

$0.001$ to the base $10$
So, as per 
logarithm,
$\log _{ 10 }{ 0.001 } =\log _{ 10 }{ { 10 }^{ -3 } } $
$\log _{ 10 }{ 0.001 } =-3\log _{ 10 }{ { 10 } } $
$\log _{ 10 }{ 0.001 } =-3\times 1$
$\log _{ 10 }{ 0.001 } =-3$

$\log V = 2 \log 2 - \log 3 + \log \pi + 3 \log r$ can be expressed as

  1. $V = \dfrac{4}{3} \pi r^{3}$

  2. $ V = \dfrac{2}{3} \pi r^{3}$

  3. $ V = \dfrac{4}{3} \pi r$

  4. $ V = \dfrac{2}{3} \pi r$


Correct Option: A
Explanation:
$ \log { V }  = 2\log { 2 } -\log { 3 } +\log { \pi  }  + 3\log { r }$
$ \log V = \log ({ 2 }^{ 2 }\times \pi \times { r }^{ 3 }) - \log { 3 }$
$ \log V = \log \dfrac { 4\pi { r }^{ 3 } }{ 3 } $
Removing log
$V = \dfrac {4}{3} \pi r^3$

Which of the following is true for $\log _25$?

  1. An integer

  2. A rational number

  3. An irrational number

  4. A whole number


Correct Option: C
Explanation:

Let us assume that $\log _{ 2 }{ 5 } =\frac { p }{ q } $ , where $p,q$ are integers

We have $5=2^{\frac{p}{q}}$
$\Rightarrow 5^{q}=2^{p}$
This suggest that $5$ and $2$ are not mutually prime , But $2$ and $5$ are mutually prime
Therefore $\log _{ 2 }{ 5 } $ is an irrational number

If $\log _{10}(x - 10) = 1$, then value of $x$ is

  1. $10$

  2. $13$

  3. $20$

  4. $26$


Correct Option: C
Explanation:

$ \log _{ 10 }{ (x-10) } =1$


$ \log _{ 10 }{ (x-10) } =\log _{ 10 }{ 10 }$

$x-10=10$

$  = 20 $

The value of $7 log _a \displaystyle \frac{16}{15} + 5 log _a \frac{25}{24} + 3 log _a \frac{81}{80}$ is

  1. $log _{a3}$

  2. $log _{a1}$

  3. $log _{a2}$

  4. $log _{a5}$


Correct Option: C
Explanation:

We have,

$7log _a\dfrac{16}{15}+5log _a\dfrac{25}{24}+3log _a\dfrac{81}{80}$
$\Rightarrow log _a(\dfrac{16}{15})^7+log _a(\dfrac{25}{24})^5+log _a(\dfrac{81}{80})^3$
$\Rightarrow log _a(\dfrac{16}{15})^7\times (\dfrac{25}{24})^5\times (\dfrac{81}{80})^3$
$\Rightarrow log _a\dfrac{16^3\times 16^4}{5^7\times 3^7}\times \dfrac{5^5\times 5^5}{8^5\times 3^5}\times \dfrac{3^6\times 3^6}{16^3\times 5^3}$
$\Rightarrow log _a\dfrac{16^4}{1\times 1}\times \dfrac{1}{8^5\times 1}\times \dfrac{1}{1}$
$\Rightarrow log _a\dfrac{2^4\times 8^4}{8^5}$
$\Rightarrow log _a\dfrac{16}{8}$
$\Rightarrow log _a2$

Hence, this is the answer.

If $log _{10} x - log _{10} \sqrt x = \displaystyle \frac{2}{log _{10} x}$, then value of x is

  1. $\displaystyle \frac{1}{100}$ or $100$

  2. $\pm$ 2

  3. 10 or $\displaystyle \frac{1}{10}$

  4. 100


Correct Option: A
Explanation:

Consider the given equation.

$log _{10}x-log _{10}\sqrt x=\dfrac{2}{log _{10}x}$
$log _{10}\dfrac{x}{\sqrt x}=\dfrac{2}{log _{10}x}$
$log _{10}\sqrt x=\dfrac{2}{log _{10}x}$
$\dfrac{1}{2}log _{10}\ x=\dfrac{2}{log _{10}x}$
$\dfrac{1}{2}(log _{10}\ x)^2=2$
$(log _{10}\ x)^2=4$
$log _{10}\ x=\pm 2$
$x=10^{\pm2}$
$x=100\ or\ \dfrac{1}{100}$

Hence, this is the answer.

If $\displaystyle \frac{log _2 (9 - 2^x)}{3 - x} = 1$, then value of x is

  1. x = 4

  2. x = + 1 or -1

  3. x = $\pm$ 2

  4. x = 0


Correct Option: D
Explanation:

We have,

$\dfrac{log _2(9-2^x)}{3-x}=1$
$log _2(9-2^x)=3-x$
$9-2^x=2^{3-x}$                $ .......... (1)$

From option $(D)$
$9-2^0=2^{3-0}$
$9-1=2^3$
$8=8$

Hence, $x=0$ is the root of this equation.

Hence, only option $D$ is correct.

The value of $\log _{ \frac{1}{2} }{ 4 } $ is

  1. $-2$

  2. $0$

  3. $\dfrac{1}{2}$

  4. $2$


Correct Option: A
Explanation:

Let $\log _{\frac{1}{2}}4 = x $

Then, $(\dfrac{1}{2})^x = 4 $

$Or, (\dfrac{1}{2})^x =2^2$

$Or, 2^{-x} = 2^2$

$x= -2$

The equation  ${ \left( \log _{ 10 }{ x+2 }  \right)  }^{ 3 }+{ \left( \log _{ 10 }{ x-1 }  \right)  }^{ 3 }={ \left( 2\log _{ 10 }{ x+1 }  \right)  }^{ 3 }$ has

  1. no natural solution

  2. two rational solutions

  3. no prime solution

  4. one irrational solution


Correct Option: B,C,D
Explanation:

Let $ \log _{ 10 }{ x+2 } =a$ and $ \log _{ 10 }{ x-1 } =b$
$\therefore a+b=2\log _{ 10 }{ x+1 } $ (from the question)
Thus, the given equation(in the question) reduces to ${a}^{3}+{b}^{3}={(a+b)}^{3}$
$\Rightarrow 3ab(a+b)=0$
$\Rightarrow a=0$ or $b=0$ or $a+b=0$
$\Rightarrow \log _{ 10 }{ x+2 }=0$  or $\log _{ 10 }{ x-1 }=0$ or $2\log _{ 10 }{ x } +1=0$
$\Rightarrow x={10}^{-2}$  or  $x=10$ or  $x={ 10 }^{ -\frac { 1 }{ 2 }  }$
Hence  $x=\left{ \dfrac { 1 }{ 100 },10 ,\dfrac { 1 }{ \sqrt { 10 }  }  \right} $

- Hide questions